Midterm Review

Ace your homework & exams now with Quizwiz!

A nurse is caring for a client who has multiple sclerosis and is receiving interferon beta-1a. The nurse should identify that which of the client statements indicate a potential adverse effects of this medication? a. My body aches all over b. I have abdominal cramping c. My hair seems to be thinning d. It hurts when I urinate

Answer: A Adverse effects include flu like symptoms such as general body and muscle aches.

A nurse on a medical-surgical unit is caring for 4 clients. Which of the following clients should the nurse monitor for crepitus? a. client w/ chest tube following pneumothorax b. client w/ acute exacerbation of Crohn's disease c. client who is postoperative following a laparoscopic appendectomy. d. client who is recovering from thyroid storm

Answer: A Crepitus, a crackling sound resulting from air trapped under the skin, can be palpated following a pneumothorax, this should be reported to provider.

A nurse is providing information to a client who has early Parkinson's disease and a new prescription for pramipexole. The nurse should instruct the client to monitor for which of the following adverse effects of this medication? A. Hallucinations B. Increased salivation C. Diarrhea D. Discoloration of urine

A. CORRECT: Pramipexole can cause hallucinations within 9 months of the initial dose and might require discontinuation.

A nurse is admitting a client who has a suspected myocardial infarction (MI) and a history of angina. Which of the following findings will help the nurse distinguish stable angina from an MI? A. Stable angina can be relieved with rest and nitroglycerin. B. The pain of an MI resolves in less than 15 min. C. The type of activity that causes an MI can be identified. D. Stable angina can occur for longer than 30 min.

A. CORRECT: Stable angina can be relieved by rest and nitroglycerin

A nurse is teaching a client who has angina how to use nitroglycerin transdermal ointment. The nurse should include which of the following instructions? A. "Remove the prior dose before applying a new dose." B. "Rub the ointment directly into your skin until it is no longer visible." C. "Cover the applied ointment with a clean gauze pad." D. "Apply the ointment to the same skin area each time."

A. CORRECT: The client should remove the prior dose before applying a new dose to prevent toxicity

A nurse is beginning a physical assessment of a client who has a new diagnosis of multiple sclerosis. Which of the following findings should the nurse expect? (Select all that apply.) A. Areas of paresthesia B. Involuntary eye movements C. Alopecia D. Increased salivation E. Ataxia

A. CORRECT: Areas of loss of skin sensation are a finding in a client who has MS. B. CORRECT: Nystagmus is a finding in a client who has MS. E. CORRECT: Ataxia occurs in the client who has MS as muscle weakness develops and there is loss of coordination.

A nurse is caring for a client who asks why the provider prescribed a daily aspirin. Which of the following responses should the nurse make? A. "Aspirin reduces the formation of blood clots that could cause a heart attack." B. "Aspirin relieves the pain due to myocardial ischemia." C. "Aspirin dissolves clots that are forming in your coronary arteries." D. "Aspirin relieves headaches that are caused by other medications."

A. CORRECT: Aspirin decreases platelet aggregation that can cause a myocardial infarction.

A nurse is teaching a client who has a new prescription for nitroglycerin transdermal patch for angina pectoris. Which of the following instructions should the nurse include? A. Remove the patch each evening. B. Cut each patch in half if angina attacks are under control. C. Take off the nitroglycerin patch for 30 min if a headache occurs. D. Apply a new patch every 48 hr

A. CORRECT: In order to prevent tolerance to nitroglycerin, the client should remove the patch for 10 to 12 hr during each 24hr period.

A nurse is reinforcing teaching with a client who has Parkinson's disease and has a new prescription for bromocriptine. Which of the following instructions should the nurse include? A. Rise slowly when standing. B. Expect urine to become dark colored. C. Avoid foods containing tyramine. D. Report any skin discoloration.

A. CORRECT: Orthostatic hypotension is a common adverse effect of bromocriptine, a dopamine receptor agonist. Therefore, rising slowly when standing up will decrease the risk of dizziness and lightheadedness.

A nurse is preparing to care for a client following chest tube placement. Which of the following items should be available in the client's room? (Select all that apply.) A. Oxygen B. Sterile water C. Enclosed hemostat clamps D. Indwelling urinary catheter E. Occlusive dressing

A. CORRECT: Oxygen should be readily available in case the client develops respiratory distress following chest tube placement. The nurse should monitor respiration, oxygen saturation, and lung sounds. B. CORRECT: If the chest tubing becomes disconnected, the end of the tubing should be placed in sterile water to restore the water seal. C. CORRECT: Hemostat clamps should be available for the nurse to use to check for air leaks. E. CORRECT: If the chest tubing becomes disconnected, the nurse should immediately place a gauze dressing over the site. An occlusive dressing can also be necessary to prevent the development of a pneumothorax

A nurse is planning care for a client following the insertion of a chest tube and drainage system. Which of the following should be included in the plan of care? (Select all that apply.) A. Encourage the client to cough and deep breathe. B. Check for continuous bubbling in the suction chamber. C. Strip the drainage tubing every 4 hr. D. Clamp the tube once a day. E. Obtain a chest x-ray.

A. CORRECT: The nurse should instruct the client to cough and deep breathe. This promotes oxygenation and lung re-expansion. B. CORRECT: The nurse should check for continuous bubbling in the suction chamber to verify that suction is being maintained at an appropriate level. E. CORRECT: A chest x-ray is obtained following the procedure to verify chest tube placement.

Which menu choice by the patient with diverticulosis is best for preventing diverticulitis? a. Navy bean soup and vegetable salad b. Whole grain pasta with tomato sauce c. Baked potato with low-fat sour cream d. Roast beef sandwich on whole wheat bread

ANS: A A diet high in fiber and low in fats and red meat is recommended to prevent diverticulitis. Although all the choices have some fiber, the bean soup and salad will be the highest in fiber and the lowest in fat.

Which action would the nurse plan when admitting a patient with acute diverticulitis plan for initial care? a. Administer IV fluids. b. Prepare for colonoscopy. c. Encourage a high-fiber diet. d. Give stool softeners and enemas.

ANS: A A patient with acute diverticulitis will be NPO and given parenteral fluids. A diet high in fiber and fluids will be implemented before discharge. Bulk-forming laxatives, rather than stool softeners, are usually given. These will be implemented later in the hospitalization. The patient with acute diverticulitis will not have enemas or a colonoscopy because of the risk for perforation and peritonitis.

The nurse is caring for a patient who has a right-sided chest tube after a right lower lobectomy. Which nursing action can the nurse delegate to the unlicensed assistive personnel (AP)? a. Document the amount of drainage every 8 hours. b. Obtain samples of drainage for culture from the system. c. Assess patient pain level associated with the chest tube. d. Check the water-seal chamber for the correct fluid level.

ANS: A AP education includes documentation of intake and output. The other actions are within the scope of practice and education of licensed nursing personnel.

The nurse is caring for a patient who has a right-sided chest tube after a right lower lobectomy. Which nursing action could the nurse delegate to the assistive personnel (AP)? a. Document the amount of drainage every 8 hours. b. Obtain samples of drainage for culture from the system. c. Assess patient pain level associated with the chest tube. d. Check the water-seal chamber for the correct fluid level.

ANS: A AP education includes documentation of intake and output. The other actions are within the scope of practice and education of licensed nursing personnel.

A patient with a new ileostomy asks how much it will drain after the bowel has adapted in a few months. How many cups of drainage per day would the nurse tell the patient to expect? a. 2 b. 3 c. 4 d. 5

ANS: A After the proximal small bowel adapts to reabsorb more fluid, the average amount of ileostomy drainage is about 500 mL daily. One cup is about 240 mL.

After change-of-shift report, which patient would the nurse assess first? a. Patient with myasthenia gravis who is reporting increased muscle weakness b. Patient with a bilateral headache described as "like a band around my head" c. Patient with seizures who is scheduled to receive a dose of phenytoin (Dilantin) d. Patient with Parkinson's disease who has developed cogwheel rigidity of the arms

ANS: A Because increased muscle weakness may indicate the onset of a myasthenic crisis, the nurse would assess this patient first. The other patients would be assessed but do not appear to need immediate nursing assessments or actions to prevent life-threatening complications.

Which action will the nurse include in the plan of care for a 25-yr-old male patient with a new diagnosis of irritable bowel syndrome (IBS)? a. Encourage the patient to express concerns and ask questions about IBS. b. Suggest that the patient increase the intake of milk and other dairy products. c. Teach the patient to avoid using nonsteroidal antiinflammatory drugs (NSAIDs). d. Teach the patient about the use of alosetron (Lotronex) to reduce IBS symptoms.

ANS: A Because psychologic and emotional factors can affect the symptoms for IBS, encouraging the patient to discuss emotions and ask questions is an important intervention. Alosetron has serious side effects and is used only for female patients who have not responded to other therapies. Although yogurt may be beneficial, milk is avoided because lactose intolerance can contribute to symptoms in some patients. NSAIDs can be used by patients with IBS.

A patient who developed chest pain 4 hours ago may be having a myocardial infarction. Which laboratory test result would be most helpful in indicating myocardial damage? a. Troponins b. Myoglobin c. Homocysteine (Hcy) d. Creatine kinase-MB (CK-MB)

ANS: A Cardiac troponins start to elevate 4 to 6 hours after myocardial injury and are highly specific to myocardium. They are the preferred diagnostic marker for myocardial infarction. High-sensitivity troponin (hs-cTnT, hs-cTnI) assays provide even earlier detection of a heart event, within 1-3 hours. Myoglobin rises in response to myocardial injury within 30 to 60 minutes but is nonspecific and rapidly cleared from the body, limiting its use in the diagnosis of myocardial infarction. Creatine kinase (CK-MB) increases 4 to 6 hours after myocardial injury but is rarely used now for diagnosis of acute MI. Homocysteine (Hcy) is an amino acid that is made during protein catabolism. Elevated levels of Hcy are linked to a higher risk of CVD, peripheral vascular disease, and stroke.

A female patient who had a stroke 24 hours ago has expressive aphasia. Which intervention would the nurse use to help the patient communicate? a. Ask questions that the patient can answer with ―yes or ―no. b. Develop a list of words that the patient can read and practice reciting c. Have the patient practice her facial and tongue exercises with a mirror. d. Prevent embarrassing the patient by answering for her if she does not respond.

ANS: A Communication will be facilitated and less frustrating to the patient when questions that require a ―yes or ―no response are used. When the language areas of the brain are injured, the patient might not be able to read or recite words, which will frustrate the patient without improving communication. Expressive aphasia is caused by damage to the language areas of the brain, not by the areas that control the motor aspects of speech. The nurse should allow time for the patient to respond.

A young woman with Crohn's disease develops a fever and symptoms of a urinary tract infection (UTI) with tan, fecal-smelling urine. Which information will the nurse add to a teaching plan about UTIs for this patient that goes beyond a general teaching plan for UTIs? a. Fistulas can form between the bowel and bladder. b. Bacteria in the perianal area can enter the urethra. c. Drink adequate fluids to maintain normal hydration. d. Empty the bladder before and after sexual intercourse.

ANS: A Fistulas between the bowel and bladder occur in Crohn's disease and can lead to UTI. Teaching for UTI prevention in general includes good hygiene, adequate fluid intake, and voiding before and after intercourse.

After having frequent diarrhea and a weight loss of 10 lb (4.5 kg) over 2 months, a patient has a new diagnosis of Crohn's disease. What would the nurse plan to teach the patient? a. Medication use b. Fluid restriction c. Enteral nutrition d. Activity restrictions

ANS: A Medications are used to induce and maintain remission in patients with inflammatory bowel disease (IBD). Decreased activity level is indicated only if the patient has severe fatigue and weakness. Fluids are needed to prevent dehydration. There is no advantage to enteral feedings if the patient is able to eat.

In preparation for discharge, the nurse teaches a patient with chronic stable angina how to use the prescribed short-acting and long-acting nitrates. Which patient statement indicates that the teaching has been effective? a. I will sit down before I put the nitroglycerin under my tongue. b. I will check my pulse rate before I take any nitroglycerin tablets. c. I will put the nitroglycerin patch on as soon as I get any chest pain. d. I will remove the nitroglycerin patch before taking sublingual nitroglycerin.

ANS: A The patient should sit down before taking the nitroglycerin to decrease cardiac workload and prevent orthostatic hypotension. Transdermal nitrates are used prophylactically rather than to treat acute pain and can be used concurrently with sublingual nitroglycerin. Although the nurse should check blood pressure before giving nitroglycerin, patients do not need to check the pulse rate before taking nitrates.

The nurse on the intermediate care unit received change-of-shift report on four patients with hypertension. Which patient would the nurse assess first? a. 48-yr-old with a BP of 160/92 mm Hg who reports chest pain b. 50-yr-old with a BP of 190/104 mm Hg whose creatinine is 1.7 mg/dL c. 52-yr-old with a BP of 198/90 mm Hg who has intermittent claudication d. 43-yr-old with a BP of 172/98 mm Hg whose urine shows microalbuminuria

ANS: A The patient with chest pain may be experiencing acute myocardial infarction and rapid assessment and intervention are needed.

Which patient would the nurse assess first after receiving change-of-shift report? a. A 40-yr-old patient who has a distended abdomen and tachycardia b. A 60-yr-old patient whose ileostomy has drained 800 mL over 8 hours c. A 30-yr-old patient with ulcerative colitis who had six liquid stools in 4 hours d. A 50-yr-old patient with familial adenomatous polyposis who has occult blood in the stool

ANS: A The patient's abdominal distention and tachycardia suggest hypovolemic shock caused by problems such as peritonitis or intestinal obstruction, which will require rapid intervention. The other patients would be assessed as quickly as possible, but the data do not indicate any life-threatening complications associated with their diagnoses.

Two days after an acute myocardial infarction (MI), a patient reports stabbing chest pain that increases with a deep breath. Which action will the nurse take first? a. Auscultate the heart sounds. b. Check the patient's temperature. c. Give PRN acetaminophen (Tylenol). d. Notify the patient's health care provider.

ANS: A The patient's clinical manifestations and history are consistent with pericarditis, and the first action by the nurse would be to listen for a pericardial friction rub. Checking the temperature and notifying the health care provider are also appropriate actions but would not be done before listening for a rub. Acetaminophen (Tylenol) is not effective for pericarditis pain. An analgesic would not be given before assessment of a new symptom.

After having a myocardial infarction (MI) and successful percutaneous coronary intervention, the patient states, "It was just a little chest pain. As soon as I get out of here, I'm going for my vacation as planned." Which reply would be most appropriate for the nurse to make? a. What do you think caused your chest pain? b. Where are you planning to go for your vacation? c. Sometimes plans need to change after a heart attack. d. Recovery from a heart attack takes at least a few weeks.

ANS: A When the patient is experiencing denial, the nurse should assist the patient in testing reality until the patient has progressed beyond this step of the emotional adjustment to MI. Asking the patient about vacation plans reinforces the patient's plan, which is not appropriate in the immediate post-MI period. Reminding the patient in denial about the MI is likely to make the patient angry and lead to distrust of the nursing staff.

After receiving change-of-shift report on the following four patients, which patient would the nurse see first? a. A 60-yr-old patient with right-sided weakness who has an infusion of tPA prescribed b. A 50-yr-old patient who has atrial fibrillation and a new order for warfarin (Coumadin) c. A 30-yr-old patient with a subarachnoid hemorrhage 2 days ago who has nimodipine scheduled d. A 40-yr-old patient who had a transient ischemic attack yesterday and has a dose of aspirin due

ANS: A tPA needs to be infused within the first few hours after stroke symptoms start to be effective in minimizing brain injury. The other medications should also be given as quickly as possible, but timing of the medications is not as critical.

A patient with Parkinson's disease is admitted to the hospital for treatment of pneumonia. Which interventions would the nurse include in the plan of care? (Select all that apply.) a. Provide an elevated toilet seat. b. Cut patient's food into small pieces. c. Serve high-protein foods at each meal. d. Place an armchair at the patient's bedside. e. Observe for sudden exacerbation of symptoms

ANS: A, B, D Because the patient with Parkinson's disease has difficulty chewing, food would be cut into small pieces. An armchair would be used when the patient is seated so that the patient can use the arms to assist with getting up from the chair. An elevated toilet seat will facilitate getting on and off the toilet. High-protein foods will decrease the effectiveness of L-dopa; limiting protein intake to the evening meal can decrease this problem. Parkinson's disease is a steadily progressive disease without acute exacerbations.

A patient in the clinic reports a recent episode of dysphasia and left-sided weakness at home that resolved after 2 hours. Which medication topic would the nurse anticipate teaching the patient? a. tPA b. Aspirin c. Warfarin d. Nimodipine

ANS: B After a transient ischemic attack, patients typically are started on medications such as aspirin to inhibit platelet function and decrease stroke risk. tPA is used for acute ischemic stroke. Warfarin is usually used for patients with atrial fibrillation. Nimodipine is used to prevent cerebral vasospasm after a subarachnoid hemorrhage

A patient who has recently had an acute myocardial infarction (AMI) ambulates in the hospital hallway. Which data would indicate to the nurse that the patient should stop and rest? a. O2 saturation drops from 99% to 95%. b. Heart rate increases from 66 to 98 beats/min. c. Respiratory rate goes from 14 to 20 breaths/min. d. Blood pressure (BP) changes from 118/60 to 126/68 mm Hg.

ANS: B A change in heart rate of more than 20 beats over the resting heart rate indicates that the patient should stop and rest. The increases in BP and respiratory rate, and the slight decrease in O2 saturation, are normal responses to exercise.

A patient being admitted with an acute exacerbation of ulcerative colitis reports crampy abdominal pain and passing 15 bloody stools a day. Which intervention would the nurse include in the patient's plan of care? a. Administer oral metoclopramide. b. Instruct the patient not to eat or drink. c. Administer cobalamin (vitamin B12) injections. d. Teach the patient about total colectomy surgery

ANS: B An initial therapy for an acute exacerbation of inflammatory bowel disease (IBD) is to rest the bowel by making the patient NPO. Metoclopramide increases peristalsis and will worsen symptoms. Cobalamin (vitamin B12) is absorbed in the ileum, which is not affected by ulcerative colitis. Although total colectomy is needed for some patients, there is no indication that this patient is a candidate during this acute phase.

The nurse is admitting a patient who has chest pain. Which assessment data suggest that the pain may be from an acute myocardial infarction? a. The pain increases with deep breathing. b. The pain has lasted longer than 30 minutes. c. The pain is relieved after the patient takes nitroglycerin. d. The pain is reproducible when the patient raises the arms.

ANS: B Chest pain that lasts for 20 minutes or more is characteristic of AMI. Changes in pain that occur with raising the arms or with deep breathing are more typical of musculoskeletal pain or pericarditis. Stable angina is usually relieved when the patient takes nitroglycerin.

A postoperative patient has a right-sided pleural chest tube connected to a chest drainage device. There is continuous bubbling in the suction-control chamber of the wet-suction collection device. Which action would the nurse take? a. Adjust the dial on the wall regulator. b. Continue to monitor the collection device. c. Document the presence of a large air leak. d. Notify the surgeon of a possible pneumothorax.

ANS: B Continuous bubbling is expected in the suction-control chamber of a wet-suction device and indicates that the suction-control chamber is connected to suction. An air leak would be detected in the water-seal chamber. There is no evidence of pneumothorax. Increasing or decreasing the vacuum source will not adjust the suction pressure. The amount of suction applied is regulated by the amount of water in this chamber and not by the amount of suction applied to the system.

Several weeks after a stroke, a patient has impaired awareness of bladder fullness, resulting in urinary incontinence. Which intervention would the nurse plan to begin an effective bladder training program? a. Limit fluid intake to 1200 mL daily to reduce urine volume. b. Assist the patient onto the bedside commode every 2 hours. c. Use an external catheter to protect the skin and prevent embarrassment. d. Perform intermittent catheterization after each voiding to check for residual urine.

ANS: B Developing a regular voiding schedule will prevent incontinence and may increase patient awareness of a full bladder. A 1200-mL fluid restriction may lead to dehydration. Intermittent catheterization and use of an external catheter are appropriate in the acute phase of stroke but should not be considered solutions for long-term management because of the risks for urinary tract infection and skin breakdown.

The nurse is caring for a patient who is recovering from a sudden cardiac death (SCD) event and has no evidence of an acute myocardial infarction (AMI). Which information would the nurse anticipate teaching the patient? a. Sudden cardiac death events rarely reoccur. b. Additional diagnostic testing will be required. c. Long-term anticoagulation therapy will be needed. d. Limiting physical activity will prevent future SCD events.

ANS: B Diagnostic testing (e.g., stress test, Holter monitor, electrophysiologic studies, cardiac catheterization) is used to determine the possible cause of the SCD and treatment options. SCD is likely to recur. Anticoagulation therapy will not have any effect on the incidence of SCD, and SCD can occur even when the patient is resting.

The nurse is caring for a patient who was admitted to the coronary care unit following an acute myocardial infarction (AMI) and percutaneous coronary intervention the previous day. Which information would the nurse plan to provide today? a. Typical emotional responses to AMI b. When cardiac rehabilitation will begin c. Pathophysiology of coronary artery disease d. Information regarding discharge medications

ANS: B Early after an AMI, the patient will want to know when resumption of usual activities can be expected. At this time, the patient's anxiety level or denial will interfere with good understanding of complex information such as the pathophysiology of coronary artery disease. Teaching about discharge medications should be done closer to discharge. The nurse should support the patient by decreasing anxiety with information rather than discussing the typical emotional responses to myocardial infarction.

Which information will the nurse include in teaching a patient who had a proctocolectomy and ileostomy for ulcerative colitis? a. Restrict fluid intake to prevent constant liquid drainage from the stoma. b. Use care when eating high-fiber foods to avoid obstruction of the ileum. c. Irrigate the ileostomy daily to avoid having to wear a drainage appliance. d. Change the pouch every day to prevent leakage of contents onto the skin.

ANS: B High-fiber foods are introduced gradually and should be well chewed to avoid obstruction of the ileostomy. Patients with ileostomies do not have a colon for the absorption of water; they need to take in increased amounts of fluid. The pouch should be drained frequently but is changed every 5 to 7 days. The drainage from an ileostomy is liquid and continuous, so control by irrigation is not possible.

Which recommendation would the nurse provide to a patient with myasthenia gravis (MG)? a. Anticipate the need for weekly plasmapheresis treatments. b. Complete physically demanding activities early in the day. c. Protect the extremities from injury due to poor sensory perception. d. Perform frequent weight-bearing exercise to prevent muscle atrophy.

ANS: B Muscles are generally strongest in the morning, and muscle weakness is prominent by the end of the day, so activities involving muscle activity should be scheduled early. Plasmapheresis is not routinely scheduled but is used for myasthenia crisis or for situations in which corticosteroid therapy must be avoided. There is no decrease in sensation with MG. Muscle atrophy does not occur because although muscles are weak, they are still used.

A patient with a stroke has progressively increasing weakness and decreasing level of consciousness. Which patient problem would the nurse determine has the highest priority for the patient? a. Sensory deficit b. Risk for aspiration c. Musculoskeletal problem d. Risk for impaired skin integrity

ANS: B Protection of the airway is the priority of nursing care for a patient having an acute stroke. The other diagnoses are also appropriate, but interventions to prevent aspiration are the priority at this time.

A patient with Parkinson's disease (PD) has bradykinesia. Which action would the nurse include in the plan of care? a. Instruct the patient in activities that can be done while lying or sitting. b. Suggest that the patient use the arms of the chair to help push up to standing. c. Have the patient take small steps in a straight line directly in front of the feet. d. Teach the patient to keep the feet in contact with the floor and slide them forward.

ANS: B Pushing down on the arms of the chair and placing the back legs of the chair on small (2-inch) blocks help the individual with PD to stand. The patient will be encouraged to continue exercising because this will maintain functional abilities. Maintaining a wider base of support, rather than stepping directly forward, will help with balance. The patient should lift the feet and avoid a shuffling gait.

A patient who is recovering from an acute myocardial infarction (AMI) asks the nurse about safely resuming sexual intercourse. Which response by the nurse provides the most useful information for the patient? a. Most patients are able to enjoy intercourse without any complications. b. Sexual activity uses about as much energy as climbing two flights of stairs. c. The doctor will provide sexual guidelines when your heart is strong enough. d. Holding and cuddling are good ways to maintain intimacy after a heart attack.

ANS: B Sexual activity places about as much physical stress on the cardiovascular system as moderate-energy activities, such as climbing two flights of stairs. The other responses are general statements that may be accurate, but do not provide useful guide lines for judging the physical safety of the activity.

A 74-yr-old patient is seen in the health clinic with new development of a stooped posture, shuffling gait, and pill rolling-type tremor. Which topic would the nurse anticipate explaining to the patient? a. Oral corticosteroids b. Dopaminergic drugs c. Magnetic resonance imaging (MRI) d. Electroencephalogram (EEG) testing:

ANS: B The clinical diagnosis of Parkinson's is made when tremor, rigidity, akinesia, and postural instability are present. The confirmation of the diagnosis is made on the basis of improvement when dopaminergic drugs are administered. MRI and EEG are not useful in diagnosing Parkinson's disease, and corticosteroid therapy is not used to treat it.

Which action will the nurse take to evaluate the effectiveness of IV nitroglycerin for a patient with a myocardial infarction (MI)? a. Monitor heart rate. b. Ask about chest pain. c. Check blood pressure. d. Observe for dysrhythmias.

ANS: B The goal of IV nitroglycerin administration in MI is relief of chest pain by improving the balance between myocardial oxygen supply and demand. The nurse will also monitor heart rate and blood pressure and observe for dysrhythmias, but these parameters will not indicate whether the medication is effective.

After a thymectomy, a patient with myasthenia gravis receives the usual dose of pyridostigmine (Mestinon). An hour later, the patient reports diarrhea and severe abdominal cramps. Which action would the nurse take first? a. Auscultate the patient's bowel sounds. b. Notify the patient's health care provider. c. Administer the prescribed PRN antiemetic drug. d. Give the scheduled dose of prednisone (Deltasone).

ANS: B The patient's history and symptoms indicate a possible cholinergic crisis. The health care provider would be notified immediately, and it is likely that atropine will be prescribed. The other actions will be appropriate if the patient is not experiencing a cholinergic crisis.

The nurse is assessing a patient who has been admitted to the intensive care unit (ICU) with a hypertensive emergency. Which finding is most urgent to report to the health care provider? a. Urine output over 8 hours is 250 mL less than the fluid intake. b. The patient cannot move the left arm and leg when asked to do so. c. Tremors are noted in the fingers when the patient extends the arms. d. The patient reports a headache with pain at level 7 of 10 (0 to 10 scale).

ANS: B The patient's inability to move the left arm and leg indicates that a stroke may be occurring and will require immediate action to prevent further neurologic damage. The other clinical manifestations are also likely caused by the hypertension and will require rapid nursing actions, but they do not require action as urgently as the neurologic changes.

A patient who had a transverse colectomy for diverticulosis 18 hours ago has nasogastric suction. The patient reports anxiety and incisional pain. The patient's respiratory rate is 32 breaths/min, and the arterial blood gases (ABGs) indicate respiratory alkalosis with a normal arterial oxygen level. Which action would the nurse take first? a. Check to make sure the nasogastric tube is patent. b. Give the patient the PRN IV morphine sulfate 4 mg.

ANS: B The patient's respiratory alkalosis is likely caused by the increased respiratory rate associated with pain and anxiety. The nurse's first action would be to medicate the patient for pain. The health care provider may be notified about the ABGs but is likely to instruct the nurse to medicate for pain. The patient will not be able to take slow, deep breaths when experiencing pain. Checking the nasogastric tube can wait until the patient has been medicated for pain.

The health care provider is considering the use of sumatriptan (Imitrex) for a 54-yr-old male patient with migraine headaches. Which information obtained by the nurse is most important to report to the health care provider? a. The patient drinks 1 to 2 cups of coffee daily. b. The patient had a recent acute myocardial infarction. c. The patient has had migraine headaches for 30 years. d. The patient has taken topiramate (Topamax) for 2 months.

ANS: B Triptans cause coronary artery vasoconstriction and are contraindicated in patients with coronary artery disease. The other information will be reported to the health care provider, but none of it indicates that sumatriptan would be an inappropriate treatment.

A 22-yr-old female patient with an exacerbation of ulcerative colitis is having 15 to 20 stools daily and has excoriated perianal skin. Which patient behavior indicates that the nurse's teaching about skin integrity has been effective? a. The patient uses incontinence briefs to contain loose stools. b. The patient uses witch hazel compresses to soothe irritation. c. The patient asks for antidiarrheal medication after each stool. d. The patient cleans the perianal area with soap after each stool.

ANS: B Witch hazel compresses are suggested to reduce anal irritation and discomfort. Incontinence briefs may trap diarrhea and increase the incidence of skin breakdown. Antidiarrheal medications cannot be given 15 to 20 times a day. The perianal area should be washed with plain water or pH balanced cleanser after each stool.

The nurse is caring for a patient who has been experiencing stroke symptoms for 60 minutes. Which action can the nurse delegate to a licensed practical/vocational nurse (LPN/VN)? a. Assess the patient's gag and cough reflexes. b. Determine when the stroke symptoms began. c. Administer the prescribed short-acting insulin. d. Infuse the prescribed IV metoprolol (Lopressor).

ANS: C Administration of subcutaneous medications is included in LPN/VN education and scope of practice. The other actions require more education and scope of practice and should be done by the registered nurse (RN).

A patient who has a history of a transient ischemic attack (TIA) has an order for aspirin 160 mg daily. The patient says, "I don't need the aspirin today. I don't have a fever." Which action would the nurse take? a. Document that the patient refused the aspirin. b. Tell the patient that the aspirin is used to prevent a fever. c. Explain that the aspirin is ordered to decrease stroke risk. d. Call the health care provider to clarify the medication order.

ANS: C Aspirin is ordered to prevent stroke in patients who have experienced TIAs. Documentation of the patient's refusal to take the medication without providing more information is an inadequate response by the nurse. There is no need to clarify the order with the health care provider. The aspirin is not ordered to prevent aches and pains.

. A patient with a stroke is unconscious and unresponsive to stimuli. After learning that the patient has a history of gastroesophageal reflux disease (GERD), which assessment would the nurse plan to make more frequently than is routine? a. Apical pulse b. Bowel sounds c. Breath sounds d. Abdominal girth

ANS: C Because GERD may cause aspiration, the unconscious patient is at risk for developing aspiration pneumonia. Bowel sounds, abdominal girth, and apical pulse will not be affected by the patient's stroke or GERD and do not require more frequent monitoring than the routine.

A patient admitted with possible stroke has been aphasic for 3 hours and has a current blood pressure (BP) of 174/94 mm Hg. Which order by the health care provider would the nurse question? a. Keep head of bed elevated at least 30 degrees. b. Infuse normal saline intravenously at 75 mL/hr. c. Start a labetalol drip to keep BP less than 140/90 mm Hg. d. Begin tissue plasminogen activator (tPA) intravenously per protocol.

ANS: C Because elevated BP may be a protective response to maintain cerebral perfusion, antihypertensive therapy is recommended only if mean arterial pressure (MAP) is greater than 130 mm Hg or systolic pressure is greater than 220 mm Hg. Fluid intake should be 1500 to 2000 mL/day to maintain cerebral blood flow. The head of the bed should be elevated to at least 30 degrees unless the patient has symptoms of poor tissue perfusion. tPA may be administered if the patient meets the other criteria for tPA use.

Which assessment would the nurse identify as most important regarding a patient who has myasthenia gravis? a. Pupil size b. Grip strength c. Respiratory effort d. Level of consciousness

ANS: C Because respiratory insufficiency may be life threatening, it will be most important to monitor respiratory function. The other data also will be assessed but are not as critical.

Which diet choice by the patient with an acute exacerbation of inflammatory bowel disease (IBD) indicates a need for more teaching? a. Scrambled eggs b. White toast and jam c. Oatmeal with cream d. Pancakes with syrup

ANS: C During acute exacerbations of IBD, the patient would avoid high-fiber foods such as whole grains. High-fat foods also may cause diarrhea in some patients. The other choices are low residue and would be appropriate for this patient.

Which intervention would the nurse include in the plan of care for a patient with new right-sided homonymous hemianopsia after a stroke? a. Apply an eye patch to the right eye. b. Approach the patient from the right side. c. Place needed objects on the patient's left side. d. Teach the patient that the left visual deficit will resolve.

ANS: C During the acute period, the nurse would place objects on the patient's unaffected side. Because there is a visual defect in the right half of each eye, an eye patch is not appropriate. The patient would be approached from the left side. The visual deficit may not resolve, although the patient can learn to compensate for the defect.

Which stroke risk factor for a 48-yr-old male patient in the clinic is most important for the nurse to address? a. The patient is 25 pounds above the ideal weight. b. The patient drinks a glass of red wine with dinner daily. c. The patient's usual blood pressure (BP) is 170/94 mm Hg. d. The patient works at a desk and relaxes by watching television.

ANS: C Hypertension is the single most important modifiable risk factor. People who drink more than 1 (for women) or 2 (for men) alcoholic beverages a day may increase their risk for stroke. Physical inactivity and obesity contribute to stroke risk but not as much as hypertension.

A patient is diagnosed with moderate dementia after having multiple strokes. Which assessment finding would the nurse expect? a. Excessive nighttime sleepiness. b. Difficulty eating and swallowing. c. Loss of recent and long-term memory. d. Fluctuating ability to perform simple tasks.

ANS: C Loss of both recent and long-term memory is characteristic of moderate dementia. Patients with dementia have frequent nighttime awakening. Dementia is progressive, and the patient's ability to perform tasks would not have periods of improvement. Difficulty eating and swallowing is characteristic of severe dementia.

A patient with left-sided weakness that began 60 minutes earlier is admitted to the emergency department. Which prescribed diagnostic test would be done first? a. Complete blood count (CBC) b. Chest radiograph (chest x-ray) c. Computed tomography (CT) scan d. 12-Lead electrocardiogram (ECG)

ANS: C Rapid screening with a noncontrast CT scan is needed before administration of tissue plasminogen activator (tPA), which must be given within 4.5 hours of the onset of clinical manifestations of the stroke. The sooner the tPA is given, the less brain injury. The other diagnostic tests give information about possible causes of the stroke and do not need to be completed as urgently as the CT scan.

The nurse monitors a patient in the emergency department after chest tube placement for a hemopneumothorax. Which assessment finding is of most concern? a. A large air leak in the water-seal chamber b. Report of pain with each deep inspiration c. 400 mL of blood in the collection chamber d. Subcutaneous emphysema at the insertion site

ANS: C The large amount of blood may indicate that the patient is in danger of developing hypovolemic shock. An air leak would be expected after chest tube placement for a pneumothorax. Initially, brisk bubbling of air occurs in this chamber when a pneumothorax is evacuated. The pain would be treated but is not as urgent a concern as the possibility of continued hemorrhage. Subcutaneous emphysema would be monitored but is not unusual in a patient with pneumothorax. A small amount of subcutaneous air is harmless and will be reabsorbed.

A patient will attempt oral feedings for the first time after having a stroke. After assessing the gag reflex, which action would the nurse take? a. Order a varied pureed diet. b. Assess the patient's appetite. c. Assist the patient into a chair. d. Offer the patient a sip of juice.

ANS: C The patient should be as upright as possible before attempting to feed to make swallowing easier and decrease aspiration risk. To assess swallowing ability, the nurse should initially offer water or ice to the patient. Pureed diets are not recommended because the texture is too smooth. The patient may have a poor appetite, but the oral feeding should be attempted.

Which concern would the nurse anticipate for a patient who had a right hemisphere stroke? a. Right-sided hemiplegia b. Speech-language deficits c. Denial of deficits and impulsiveness d. Depression and distress about disability

ANS: C The patient with right-sided brain damage typically denies any deficits and has poor impulse control, leading to risk for injury when the patient attempts activities such as transferring from a bed to a chair. Right-sided brain damage causes left hemiplegia. Left-sided brain damage typically causes language deficits. Left-sided brain damage is associated with depression and distress about the disability.

A patient recovering from a myocardial infarction (MI) develops chest pain on day 3 that increases when taking a deep breath and is relieved by leaning forward. Which action would the nurse take as focused follow-up on this symptom? a. Assess both feet for pedal edema. b. Palpate the radial pulses bilaterally. c. Auscultate for a pericardial friction rub. d. Check the heart monitor for dysrhythmias.

ANS: C The patient's symptoms are consistent with the development of pericarditis, a possible complication of MI. The other assessments listed are not consistent with the description of the patient's symptoms.

A 72-yr-old patient with dehydration caused by an exacerbation of ulcerative colitis is receiving 5% dextrose in normal saline at 125 mL/hour. Which assessment finding by the nurse is most important to report to the health care provider? a. Skin is dry with tenting and poor turgor. b. Patient has not voided for the last 2 hours. c. Crackles are heard halfway up the posterior chest. d. Patient has had 5 loose stools over the previous 6 hours

ANS: C The presence of crackles in an older patient receiving IV fluids at a high rate suggests volume overload and a need to reduce the rate of the IV infusion. The other data will be reported but are consistent with the patient's age and diagnosis and do not require a change in the prescribed treatment

A patient who has chest pain is admitted to the emergency department (ED), and all of the following items are prescribed. Which one would the nurse arrange to be completed first? a. Chest x-ray b. Troponin level c. Electrocardiogram (ECG) d. Insertion of a peripheral IV

ANS: C The priority for the patient is to determine whether an acute myocardial infarction (AMI) is occurring so that the appropriate therapy can begin as quickly as possible. ECG changes occur very rapidly after coronary artery occlusion, and an ECG should be obtained as soon as possible. Troponin levels will increase after about 3 hours. Data from the chest x-ray may impact the patient's care but are not helpful in determining whether the patient is experiencing a myocardial infarction. Peripheral IV access will be needed but not before the ECG.

Which patient problem would the nurse identify as of highest priority for a patient who has Parkinson's disease and is unable to move the facial muscles? a. Activity intolerance b. Negative self-image c. Musculoskeletal problem d. Nutritionally compromised

ANS: D The data about the patient indicate that poor nutrition will be a concern because of decreased swallowing. The other diagnoses may also be appropriate for a patient with Parkinson's disease, but the data do not indicate that they are current problems for this patient.

A patient is being admitted with a possible stroke. Which information from the nursing assessment indicates that the patient is more likely to be having a hemorrhagic stroke than a thromboembolic stroke? a. The patient has intermittent bouts of atrial fibrillation. b. The patient has had brief episodes of right-sided hemiplegia. c. The patient has a history of treatment for infective endocarditis. d. The patient reports that the symptoms began with a severe headache.

ANS: D A sudden onset headache is typical of a subarachnoid hemorrhage. Atrial fibrillation and infective endocarditis are a risk factors for thrombotic or embolic stroke. Brief episodes of right-sided hemiplegia are consistent with transient ischemic attack and risk for embolic stroke

The emergency department nurse notes tachycardia and absent breath sounds over the right thorax of a patient who has just arrived after an automobile accident. For which intervention will the nurse prepare the patient? a. Emergency pericardiocentesis b. Stabilization of the chest wall c. Bronchodilator administration d. Chest tube connected to suction

ANS: D The patient's history and absent breath sounds suggest a right-sided pneumothorax or hemothorax, which will require treatment with a chest tube and drainage to suction. The other therapies would be appropriate for an acute asthma attack, flail chest, or cardiac tamponade, but the patient's clinical manifestations are not consistent with these problems.

Following an acute myocardial infarction, a previously healthy 63-yr-old develops heart failure. Which medication topic would the nurse anticipate including in discharge teaching? a. Calcium channel blocker b. Selective SA node inhibitor c. Digoxin and potassium therapy regimen d. Angiotensin-converting enzyme (ACE) inhibitor

ANS: D ACE inhibitor therapy is currently recommended to prevent the development of heart failure in patients who have had a myocardial infarction and as a first-line therapy for patients with chronic heart failure. Digoxin therapy for heart failure is no longer considered a first-line measure, and digoxin is added to the treatment protocol when therapy with other drugs such as ACE-inhibitors, diuretics, and -adrenergic blockers is insufficient. Calcium channel blockers are not generally used in the treatment of heart failure. Ivabradine would likely be used for a patient with HF who has symptoms despite optimal doses of other medications.

Which finding is likely in the nurse's assessment of a patient who has a large bowel obstruction? a. Referred back pain b. Metabolic alkalosis c. Projectile vomiting d. Abdominal distention

ANS: D Abdominal distention is seen in lower intestinal obstruction. Referred back pain is not a common clinical manifestation of intestinal obstruction. Metabolic alkalosis is common in high intestinal obstruction because of the loss of HCl acid from vomiting. Projectile vomiting is associated with higher intestinal obstruction.

Which patient statement would help the nurse confirm the previous diagnosis of chronic stable angina? a. The pain wakes me up at night. b. The pain is level 3 to 5 (0 to 10 scale). c. The pain has gotten worse over the last week. d. The pain goes away with a nitroglycerin tablet.

ANS: D Chronic stable angina is typically relieved by rest or nitroglycerin administration. The level of pain is not a consistent indicator of the type of angina. Pain occurring at rest or with increased frequency is typical of unstable angina.

A patient is being treated with carbidopa/levodopa (Sinemet) for Parkinson's disease. Which assessment finding would indicate to the nurse that a change in the medication or dosage may be needed? a. Shuffling gait b. Tremor at rest c. Cogwheel rigidity of limbs d. Uncontrolled head movement

ANS: D Dyskinesia is an adverse effect of the Sinemet, indicating a need for a change in medication or decrease in dose. The other findings are typical with Parkinson's disease.

Which question from the nurse would help determine if a patient's abdominal pain might indicate irritable bowel syndrome (IBS)? a. Have you been passing a lot of gas? b. What foods affect your bowel patterns? c. Do you have any abdominal distention? d. How long have you had abdominal pain?

ANS: D One criterion for the diagnosis of irritable bowel syndrome is the presence of abdominal discomfort or pain for at least 3 months. Abdominal distention, flatulence, and food intolerance are associated with IBS but are not diagnostic criteria.

When monitoring the effectiveness of treatment for a patient with a large anterior wall myocardial infarction, which is the most pertinent measurement for the nurse to obtain? a. Central venous pressure (CVP) b. Systemic vascular resistance (SVR) c. Pulmonary vascular resistance (PVR) d. Pulmonary artery wedge pressure (PAWP)

ANS: D PAWP reflects left ventricular end diastolic pressure (or left ventricular preload) and is a sensitive indicator of cardiac function. Because the patient is high risk for left ventricular failure, the PAWP must be monitored. An increase will indicate left ventricular failure. The other values would also provide useful information, but the most definitive measurement of changes in cardiac function is the PAWP.

A patient with a left-brain stroke suddenly bursts into tears when family members visit. How would the nurse respond? a. Use a calm voice to ask the patient to stop the crying behavior. b. Explain to the family that depression is normal following a stroke. c. Have the family members leave the patient alone for a few minutes. d. Teach the family that emotional outbursts are common after strokes.

ANS: D Patients who have left-sided brain stroke are prone to emotional outbursts that are not necessarily related to the emotional state of the patient. Depression after a stroke is common, but the suddenness of the patient's outburst suggests that depression is not the major cause of the behavior. The family should stay with the patient. The crying is not within the patient's control. Asking the patient to stop will lead to embarrassment.

A patient being admitted with a stroke has right-sided facial drooping and right-sided arm and leg paralysis. Which other finding would the nurse expect? a. Impulsive behavior b. Right-sided neglect c. Hyperactive left-sided tendon reflexes d. Difficulty comprehending instructions

ANS: D Right-sided paralysis indicates a left-brain stroke, which is also associated with difficulty in comprehension and use of language: the left hemisphere is dominant for language skills in right-handed persons and in most left-handed persons. Impulsive behavior and neglect are more likely with a right-side stroke. The left-side reflexes are likely to be intact.

The nurse obtains a rhythm strip on a patient who has had a myocardial infarction and makes the following analysis: no visible P waves, PR interval not measurable, ventricular rate of 162, R-R interval regular, QRS complex wide and distorted, and QRS duration of 0.18 second. How would the nurse interpret this cardiac rhythm? a. Atrial flutter b. Sinus tachycardia c. Ventricular fibrillation d. Ventricular tachycardia

ANS: D The absence of P waves, wide QRS, rate greater than 150 beats/min, and the regularity of the rhythm indicate ventricular tachycardia. Atrial flutter is usually regular, has a narrow QRS configuration, and has flutter waves present representing atrial activity. Sinus tachycardia has P waves. Ventricular fibrillation is irregular and does not have a consistent QRS duration.

A patient who was admitted with a myocardial infarction has a 45-second episode of ventricular tachycardia, then converts to sinus rhythm with a heart rate of 98 beats/min. Which action would the nurse take next? a. Immediately notify the health care provider. b. Document the rhythm and continue to monitor the patient. c. Prepare for synchronized cardioversion per agency protocol. d. Prepare to give IV amiodarone per agency dysrhythmia protocol.

ANS: D The burst of sustained ventricular tachycardia indicates that the patient has significant ventricular irritability, and antidysrhythmic medication administration is needed to prevent further episodes. The nurse should notify the health care provider after the medication is started. Cardioversion is not indicated given that the patient has returned to a sinus rhythm. Documentation and continued monitoring are not adequate responses to this situation.

A patient arrives in the emergency department with hemiparesis and dysarthria that started 2 hours previously. Health records show a history of several transient ischemic attacks (TIAs). Which interprofessional intervention at would the nurse anticipate for this patient? a. Surgical endarterectomy b. Transluminal angioplasty c. Intravenous heparin drip administration d. Tissue plasminogen activator (tPa) infusion

ANS: D The patient's history and clinical manifestations suggest an acute ischemic stroke, and a patient who is seen within 4.5 hours of stroke onset is likely to receive tPA (after screening with a CT scan). Heparin administration in the emergency phase is not indicated. Emergent carotid transluminal angioplasty or endarterectomy is not indicated for the patient who is having an acute ischemic stroke.

The nurse prepares a patient who has a left-sided pleural effusion for a thoracentesis. How should the nurse position the patient? a. High-Fowler's position with the left arm extended b. Supine with the head of the bed elevated 30 degrees c. On the right side with the left arm extended above the head d. Sitting upright with the arms supported on an over bed table

ANS: D The upright position with the arms supported increases lung expansion, allows fluid to collect at the lung bases, and expands the intercostal space so that access to the pleural space is easier. The other positions would increase the work of breathing for the patient and make it more difficult for the health care provider performing the thoracentesis.

A patient who has had chest pain for several hours is admitted with a diagnosis of rule out acute myocardial infarction (AMI). Which laboratory test is most specific for the nurse to monitor in determining whether the patient has had an AMI? a. Myoglobin b. Homocysteine c. C-reactive protein d. Cardiac-specific troponin

ANS: D Troponin levels increase about 4 to 6 hours after the onset of myocardial infarction (MI) and are highly specific indicators for MI. Myoglobin is released within 2 hours of MI, but it lacks specificity and its use is limited. The other laboratory data are useful in determining the patient's risk for developing coronary artery disease but are not helpful in determining whether an acute MI is in progress.

A nurse is teaching about levodopa with a family member of a client who has Parkinson's. Which of the following pieces of information should the nurse include? a. "A full therapeutic response may take several months to happen." b. "The medication should be taken with high-protein foods." c. "A full therapeutic response might cause vivid dreams." d. "The medication is given at the onset of mild symptoms."

Answer: A

A client had a new colostomy created 2 days earlier and is beginning to pass malodorous flatus from the stoma. What is the correct interpretation by the nurse? 1.This is a normal, expected event. 2.The client is experiencing early signs of ischemic bowel. 3.The client should not have the nasogastric tube removed. 4.This indicates inadequate preoperative bowel preparation.

Ans: 1 As peristalsis returns following creation of a colostomy, the client begins to pass malodorous flatus. This indicates returning bowel function and is an expected event. Within 72 hours of surgery, the client should begin passing stool via the colostomy. Options 2, 3, and 4 are incorrect interpretations.

The nurse is providing discharge teaching for a client with newly diagnosed Crohn's disease about dietary measures to implement during exacerbation episodes. Which statement made by the client indicates a need for further instruction? 1. "I should increase the fiber in my diet." 2. "I will need to avoid caffeinated beverages." 3. "I'm going to learn some stress reduction techniques." 4."I can have exacerbations and remissions with Crohn's disease."

Ans: 1 Crohn's disease is an inflammatory disease that can occur anywhere in the gastrointestinal tract but most often affects the terminal ileum and leads to thickening and scarring, a narrowed lumen, fistulas, ulcerations, and abscesses. It is characterized by exacerbations and remissions. If stress increases the symptoms of the disease, the client is taught stress management techniques and may require additional counseling. The client is taught to avoid gastrointestinal stimulants containing caffeine and to follow a high-calorie and high-protein diet. A low-fiber diet may be prescribed, especially during periods of exacerbation.

Diphenoxylate hydrochloride with atropine sulfate is prescribed for a client with ulcerative colitis. The nurse should monitor the client for which therapeutic effect of this medication? 1.Decreased diarrhea 2.Decreased cramping 3.Improved intestinal tone 4.Elimination of peristalsis

Ans: 1 Diphenoxylate hydrochloride with atropine sulfate is an antidiarrheal product that decreases the frequency of defecation, usually by reducing the volume of liquid in the stools. The remaining options are not associated therapeutic effects of this medication.

A client is admitted to the hospital with a diagnosis of acute diverticulitis. What should the nurse expect to be prescribed for this client? 1.NPO (nothing by mouth) status 2.Ambulation at least 4 times daily 3.Cholinergic medications to reduce pain 4.Coughing and deep breathing every 2 hours

Ans: 1 During the acute phase of diverticulitis, the goal of treatment is to rest the bowel and allow the inflammation to subside. The client remains NPO and is placed on bed rest. Pain occurs from bowel spasms, and increased intra-abdominal pressure (coughing and deep breathing) may precipitate an attack. Ambulation and cholinergics will increase peristalsis.

The nurse is giving dietary instructions to a client who has a new colostomy. The nurse should encourage the client to eat foods representing which diet for the first 4 to 6 weeks postoperatively? 1.Low fiber 2.Low calorie 3.High protein 4.High carbohydrate

Ans: 1 For the first 4 to 6 weeks after colostomy formation, the client should consume a low-fiber diet. After this period, the client should eat a high-carbohydrate, high-protein diet. The client also is instructed to add new foods, including those with fiber, one at a time to determine tolerance to that food.

The OR nurse is setting up a water-seal chest drainage system for a patient who has just had a thoracotomy. The nurse knows that the amount of suction in the system is determined by the water level. At what suction level should the nurse set the system? A) 20 cm H2O B) 15 cm H2O C) 10 cm H2O D) 5 cm H2O

Answer: A

The nurse is providing discharge dietary teaching to a client with a history of irritable bowel syndrome (IBS). What comment made by the client tells the nurse that further instruction is needed? 1."I'll eat more beans and peas." 2."I should eliminate caffeine and alcohol." 3."I'm afraid my son will get this disease." 4."I know I need to take vitamins and mineral supplements."

Ans: 1 IBS clients have problems with excess gas formation, with increased distention and bloating that is accompanied by rumbling abdominal sounds, belching, and flatulence, so legumes such as beans and peas should be avoided. Caffeine and alcohol also have to be eliminated. IBS can be inherited. Vitamins and mineral supplements are generally included in the dietary regime.

The nurse provides instructions to a client about measures to treat inflammatory bowel syndrome (IBS). Which statement by the client indicates a need for further teaching? 1."I need to limit my intake of dietary fiber." 2."I need to drink plenty, at least 8 to 10 cups daily." 3."I need to eat regular meals and chew my food well." 4."I will take the prescribed medications because they will regulate my bowel patterns."

Ans: 1 IBS is a functional gastrointestinal disorder that causes chronic or recurrent diarrhea, constipation, and/or abdominal pain and bloating. Dietary fiber and bulk help to produce bulky, soft stools and establish regular bowel elimination habits. Therefore, the client should consume a high-fiber diet. Eating regular meals, drinking 8 to 10 cups of liquid a day, and chewing food slowly help to promote normal bowel function. It is important to note that food and fluid tolerance is different with every client, and high fiber foods may trigger diarrhea for some. Medication therapy depends on the main symptoms of IBS. Bulk-forming laxatives or antidiarrheal agents or other agents may be prescribed.

The nurse is providing instructions to a client diagnosed with irritable bowel syndrome (IBS) who is experiencing abdominal distention, flatulence, and diarrhea. What interventions should the nurse include in the instructions? Select all that apply. 1.Eat yogurt. 2.Take loperamide to treat diarrhea. 3.Use stress management techniques. 4.Avoid foods such as cabbage and broccoli. 5.Decrease fiber intake to less than 15 g/day.

Ans: 1, 2, 3, 4 IBS is a common, chronic functional disorder, meaning that no organic cause is currently known. Treatment is directed at psychological and dietary factors and medications to regulate stool output. Options 1, 2, 3, and 4 are correct, as clients diagnosed with IBS whose primary symptoms are abdominal distention and flatulence should be advised to avoid common gas-producing foods such as broccoli and cabbage and to consume yogurt, as it may be better tolerated than milk. In addition, the probiotics found in yogurt may be beneficial because alterations in intestinal bacteria are believed to exacerbate IBS. The client should be advised to take loperamide, a synthetic opioid that slows intestinal transit and treats diarrhea when it occurs. Also, psychological stressors are associated with development and exacerbation of IBS, so stress management techniques are important. Option 5, decrease fiber intake, is incorrect, as clients should be encouraged to have a dietary fiber intake of at least 20 g/day.

A client with a history of gastrointestinal upset has been diagnosed with acute diverticulitis. The nurse should anticipate a prescription from the primary health care provider for which type of diet for this client? 1.A low-fat diet 2.A low-fiber diet 3.A high-protein diet 4.A high-carbohydrate diet

Ans: 2 A low-fiber diet places less strain on the intestines because this type of diet is easier to digest. Clients should avoid high-fiber foods when experiencing acute diverticulitis. As the attack resolves, fiber can be added gradually to the diet.

The nurse is providing care for a client with a recent transverse colostomy. Which observation requires immediate notification of the primary health care provider? 1.Stoma is beefy red and shiny 2.Purple discoloration of the stoma 3.Skin excoriation around the stoma 4.Semi-formed stool noted in the ostomy pouch

Ans: 2 Ischemia of the stoma would be associated with a dusky or bluish or purple color. A beefy red and shiny stoma is normal and expected. Skin excoriation needs to be addressed and treated but does not require as immediate attention as purple discoloration of the stoma. Semi-formed stool is a normal finding.

The nurse is providing instructions to a client with a colostomy about measures to reduce the odor from the colostomy. Which client statement indicates that the educational session was effective? 1."I should be sure to eat at least 1 cucumber every day." 2."Beet greens, parsley, or yogurt will help to control the colostomy odor." 3."I will need to increase my egg intake and try to eat ½ to 1 egg per day." 4."Green vegetables such as spinach and broccoli will prevent odor, and I should eat these foods every day."

Ans: 2 The client should be taught to include deodorizing foods in the diet, such as beet greens, parsley, buttermilk, and yogurt. Spinach also may reduce odor, but it is a gas-forming food and should be avoided. Cucumbers, eggs, and broccoli also are gas-forming foods and should be avoided or limited by the client.

The nurse is providing dietary instructions to a client with a diagnosis of irritable bowel syndrome. The nurse determines that education was effective if the client states the need to avoid which food? 1.Rice 2.Corn 3.Broiled chicken 4.Cream of wheat

Ans: 2 The client with irritable bowel should take in a diet that consists of 30 to 40 g of fiber daily because dietary fiber will help produce bulky, soft stools and establish regular bowel habits. The client should also drink 8 to 10 glasses of fluid daily and chew food slowly to promote normal bowel function. Foods that are irritating to the intestines need to be avoided. Corn is high in fiber but can be very irritating to the intestines and should be avoided. The food items in the other options are acceptable to eat.

The nurse should instruct a client with an ileostomy to include which action as part of essential care of the stoma? 1.Massage the area below the stoma. 2.Take in high-fiber foods such as nuts. 3.Limit fluid intake to prevent diarrhea. 4.Cleanse the peristomal skin meticulously.

Ans: 4 The peristomal skin must receive meticulous cleansing because ileostomy drainage has more enzymes and is more caustic to the skin than colostomy drainage. The area below the ileostomy may be massaged as needed if the ileostomy becomes blocked by high-fiber foods. Foods such as nuts and those with seeds will pass through the ileostomy. The client should be taught that these foods will remain undigested. Fluid intake should be at least 6 to 8 glasses of water per day to prevent dehydration.

The nurse caring for a client diagnosed with inflammatory bowel disease (IBD) recognizes that which classifications of medications may be prescribed to treat the disease and induce remission? Select all that apply. 1.Antidiarrheal 2.Antimicrobial 3.Corticosteroid 4.Aminosalicylate 5.Biological therapy 6.Immunosuppressant

Ans: 2, 3, 4, 5, 6 Pharmacological treatment for IBD aims to decrease the inflammation to induce and then maintain a remission. Five major classes of medications used to treat IBD are antimicrobials, corticosteroids, aminosalicylates, biological and targeted therapy, and immunosuppressants. Medications are chosen based on the location and severity of inflammation. Depending on the severity of the disease, clients are treated with either a "step-up" or "step-down" approach. The step-up approach uses less toxic therapies (e.g., aminosalicylates and antimicrobials) first, and more toxic medications (e.g., biological and targeted therapy) are started when initial therapies do not work. The step-down approach uses biological and targeted therapy first. Option 1, antidiarrheals, is incorrect. Although an antidiarrheal may be used to treat the symptoms of IBD, it does not treat the disease (the inflammation) or induce remission. In addition, antidiarrheals should be used cautiously in IBD because of the danger of toxic megacolon (colonic dilation greater than 5 cm).

The nurse is caring for a client experiencing an exacerbation of Crohn's disease. Which intervention should the nurse anticipate the primary health care provider prescribing? 1.Enteral feedings 2.Fluid restrictions 3.Oral corticosteroids 4.Activity restrictions

Ans: 3 Crohn's disease is a form of inflammatory bowel disease that is a chronic inflammation of the gastrointestinal (GI) tract. It is characterized by periods of remission interspersed with periods of exacerbation. Oral corticosteroids are used to treat the inflammation of Crohn's disease, so option 3 is the correct one. In addition to treating the GI inflammation of Crohn's disease with medications, it is also treated by resting the bowel. Therefore, option 1 is incorrect. Option 2 is incorrect, as clients with Crohn's disease typically have diarrhea and would not be on fluid restrictions. Option 4, activity restrictions, is not indicated. The client can do activities as tolerated but should avoid stress and strain.

The nurse is assisting a client with Crohn's disease to ambulate to the bathroom. After the client has a bowel movement, the nurse should assess the stool for which characteristic that is expected with this disease? 1.Blood in the stool 2.Chalky gray stool 3.Loose, watery stool 4.Dry, hard, constipated stool

Ans: 3 Crohn's disease is characterized by non-bloody diarrhea of usually not more than 4 or 5 stools daily. Over time, the episodes of diarrhea increase in frequency, duration, and severity. Options 1, 2, and 4 are not characteristics of the stool in Crohn's disease.

The nurse is caring for a hospitalized client with a diagnosis of ulcerative colitis. Which finding, if noted on assessment of the client, should the nurse report to the primary health care provider (PHCP)? 1.Hypotension 2.Bloody diarrhea 3.Rebound tenderness 4.A hemoglobin level of 12 mg/dL (120 mmol/L)

Ans: 3 Rebound tenderness may indicate peritonitis. Bloody diarrhea is expected to occur in ulcerative colitis. Because of the blood loss, the client may be hypotensive and the hemoglobin level may be lower than normal. Signs of peritonitis must be reported to the PHCP

A client has just had surgery to create an ileostomy. The nurse assesses the client in the immediate postoperative period for which most frequent complication of this type of surgery? 1.Folate deficiency 2.Malabsorption of fat 3.Intestinal obstruction 4.Fluid and electrolyte imbalance

Ans: 4 A frequent complication that occurs following ileostomy is fluid and electrolyte imbalance. The client requires constant monitoring of intake and output to prevent this from occurring. Losses require replacement by intravenous infusion until the client can tolerate a diet orally. Intestinal obstruction is a less frequent complication. Fat malabsorption and folate deficiency are complications that could occur later in the postoperative period.

The nurse is caring for a client after abdominal surgery and creation of a colostomy. The nurse is assessing the client for a prolapsed stoma and should expect to note which observation if this is present? 1.A sunken and hidden stoma 2.A narrow and flattened stoma 3.A stoma that is dusky or bluish 4.A stoma that is elongated with a swollen appearance

Ans: 4 A prolapsed stoma is one in which the bowel protrudes, causing an elongated and swollen appearance of the stoma. A retracted stoma is characterized by a sinking of the stoma. A stoma with a narrow opening is described as being stenosed. Ischemia of the stoma would be associated with a dusky or bluish color.

A client with Crohn's disease is experiencing acute pain, and the nurse provides information about measures to alleviate the pain. Which statement by the client indicates the need for further teaching? 1."I know I can massage my abdomen." 2."I will continue using antispasmodic medication." 3."One of the best things I can do is use relaxation techniques." 4."The best position for me is to lie supine with my legs straight."

Ans: 4 Pain associated with Crohn's disease is alleviated by the use of analgesics and antispasmodics and also by practicing relaxation techniques, applying local cold or heat to the abdomen, massaging the abdomen, and lying with the legs flexed. Lying with the legs extended is not useful because it increases the muscle tension in the abdomen, which could aggravate inflamed intestinal tissues as the abdominal muscles are stretched.

A client is diagnosed with a moderate case of acute ulcerative colitis. The nurse doing dietary teaching should give the client examples of foods to eat that represent which therapeutic diet? 1.High fat with milk 2.Low fiber with milk 3.High protein with milk 4.Low fiber without milk

Ans: 4 The client with a mild to moderate case of acute ulcerative colitis often is prescribed a diet that is low in fiber and does not include milk. This will help to reduce the frequency of diarrhea for this client. The remaining options are incorrect diets and may cause discomfort for the client.

A client with acute ulcerative colitis requests a snack. Which is the most appropriate snack for this client? 1.Carrots and ranch dip 2.Whole-grain cereal and milk 3.A cup of popcorn and a cola drink 4.Applesauce and a graham cracker

Ans: 4 The diet for the client with ulcerative colitis should be low fiber (low residue). The nurse should avoid providing foods such as whole-wheat grains, nuts, and fresh fruits or vegetables. Typically, lactose-containing foods also are poorly tolerated. The client also should avoid caffeine, pepper, and alcohol.

The nurse is caring for a client with ulcerative colitis. Which finding does the nurse determine is consistent with this diagnosis? 1.Hypercalcemia 2.Hypernatremia 3.Frothy, fatty stools 4.Decreased hemoglobin

Ans: 4 Ulcerative colitis is an inflammatory disease of the large colon. Findings associated with ulcerative colitis include diarrhea with up to 10 to 20 liquid bloody stools per day, weight loss, anorexia, fatigue, increased white blood cell count, increased erythrocyte sedimentation rate, dehydration, hyponatremia, and hypokalemia (not hypercalcemia). Because of the loss of blood, clients with ulcerative colitis commonly have decreased hemoglobin and hematocrit levels. Clients with ulcerative colitis have bloody diarrhea, not steatorrhea (fatty, frothy, foul-smelling stools).

A nurse is planning care for a female client who has severe irritable bowel syndrome with diarrhea (IBS-D) and a new prescription for alosetron. Which of the following interventions should the nurse include in the plan of care? a. The client must sign agreement w/ provider before beginning alosetron. b. The client must stop taking alosetron if diarrhea continues for 1 week after beginning medication. c. The client should expect to have a slower heart rate while taking alosetron. d. The client should use a barrier birth control method because alosetron interacts with oral contraceptives.

Ans: A Alosetron has potentially fatal adverse effects associated with constipation and bowel obstruction. The FDA has allowed alosetron to be placed on the market only if the clients sign and adhere to a risk management program, which includes signing a client-provider agreement before starting alosetron.

A nurse is caring for a client who has an acute exacerbation of Crohn's disease. Which of the the following actions should the nurse take? a. ensure bowel rest b. offer sparkling water c. administer a stool softener d. offer plain warm tea frequently

Ans: A Clients who have an exacerbation of Crohn's disease usually require NPO status to ensure bowel rest and promote healing and recovery

A nurse is caring for a client who has acute diverticulitis. While the client has active inflammation, the nurse should instruct which of the following foods in her diet? a. white bread and plain yogurt b. shredded wheat cereal and blueberries c. broccoli and kidney beans d. oatmeal and fresh pears

Ans: A Maintain low-fiber, low-fat

A nurse is teaching a client who has a colostomy about ways to reduce flatus and odor. Which of the following strategies should the nurse include? a. eat crackers and yogurt regularly b. chew mind hum throughout the day c. drink orange juice every day d. put an aspirin in the pouch

Ans: A helps reduce flatus, which contributes to odor

A nurse is caring for a client who has ulcerative colitis. The provider prescribes bed rest with bathroom privileges. When the client asks the nurse why he has to stay in bed, which of the following responses should the nurse provide? a. you need to conserve energy at this time b. lying quietly in bed helps slow down the activity in your intestines c. staying in bed promotes the rest and comfort you need d. staying in bed will help prevent injury and minimize your fall risk.

Ans: B The greatest risk to the client is complication from severe diarrhea such as dehydration, electrolyte imbalances, and gastrointestinal bleeding and trauma. Activity restriction can help reduce intestinal peristalsis and diarrhea.

A nurse is caring for a client who has recovered from acute diverticulitis. The nurse should instruct the client to increase his intake of which of the following foods when the inflammation subsides? a. cucumbers and tomatoes b. cabbage and peaches c. strawberries and corn d. figs and nuts

Ans: B When the acute inflammation has subsided, the client should increase his intake of foods that are high in fiber, such as wheat bran, whole-grain bread, and fresh fruits and vegetables and do not contain seeds.

A nurse is demonstrating colostomy care to a client who has a new colostomy. Which of the following actions should the nurse teach the client to perform? (SATA) a. use antimicrobial ointment on the peristomal skin b. empty the bag when it is one-third to one-half full c. cute the skin barrier opening a little larger than the ostomy d. wash the peristomal skin with mild soap and water e. apply the skin barrier while the skin is slightly moist

Ans: B C D Avoid full bag, cut opening about 1/16 to 1/8 larger than the stoma to avoid applying any constricting pressure to stoma.

A nurse is providing dietary teaching to a client who has diverticulitis about preventing acute attacks. Which of the following foods should the nurse recommend? a. foods high in vitamin b. foods low in fat c. foods high in fiber d. foods love in calories

Ans: C Long-term low-fiber eating habits and increased intracolonic pressure lead to straining during bowel movements, causing the development of diverticula. High-fiber foods help strengthen and maintain the active motility of the gastrointestinal tract.

A nurse is caring for a client who has diverticulitis and a new prescription for a low-fiber diet. Which of the following food items should the nurse remove from the client's meal tray? a. canned fruit b. white bread c. broiled hamburger d. coleslaw

Ans: D Coleslaw contains raw cabbage, which is high in fiber. Clients who are on a low-fiber diet should avoid most raw vegetables.

A nurse is caring for a client who has gastrointestinal bleeding. The provider suspects a bleeding lesion in the colon. The initial approach to treatment likely will involve which of the following procedures? a. exploratory laparotomy b. double-contract barium enema c. magnetic resonance imaging d. colonoscopy

Ans: D Identifies cause and location of bleeding.

The client has a right-sided chest tube. As the client is getting out of the bed it is acci- dentally pulled out of the pleural space. Which action should the nurse implement first? 1. Notify the health-care provider to have chest tubes reinserted STAT. 2. Instruct the client to take slow shallow breaths until the tube is reinserted. 3. Take no action and assess the client's respiratory status every 15 minutes. 4. Tape a petroleum jelly occlusive dressing on three (3) sides to the insertion site.

Answer: 4

The nurse is assisting a health care provider with the removal of a chest tube. The nurse should instruct the client to take which action? 1.Exhale slowly. 2.Stay very still. 3.Inhale and exhale quickly. 4.Perform the Valsalva maneuver.

Answer: 4 When the chest tube is removed, the client is asked to perform the Valsalva maneuver (take a deep breath, exhale, and bear down). The tube is quickly withdrawn, and an airtight dressing is taped in place. An alternative instruction is to ask the client to take a deep breath and hold the breath while the tube is removed.

A nurse is preparing to administer dantrolene to a client who has muscle spasticity. Which of the following findings from the client's medical record should the nurse identify as a contraindication to the administration of this medication? a. history of cirrhosis b. history of multiple sclerosis c. history of cerebral palsy d. history of malignant hyperthermia

Answer: A Dantrolene is contraindicated for clients who has active liver disease because it is hepatotoxic and can cause liver failure. Liver function tests are monitored for clients throughput the treatment with this medication.

In a spontaneously breathing patient, the nurse notes tidaling of the water level in the water-seal chamber of the chest tube drainage system. The nurse would: a. continue to monitor the patient b. check all connections for a leak in the system c. raise the collection unit above the level of the heart d. clamp the tubing at a distal point away from the patient.

Answer: A Tidaling of the water level in the water-seal chamber is a normal finding in a chest drainage system of a spontaneously breathing patient. Tidaling coincides with the patient's spontaneous respirations. As a result, you need only to continue to monitor the patient. You would never raise the chest drainage unit above the patient's heart, as this would not permit drainage. There are very few indications to clamp a chest tube; tidaling is not one of them. Checking all connections for a leak is part of your ongoing assessment of the patient with a chest tube.

The following would result in a loss of the water seal: A. Momentary tipping-over of the Atrium drainage system. B. Evaporation of the water in the water seal chamber below the 2 cm mark. C. Suction removed or turned off. D. The drainage chambers are full.

Answer: B

A client with multiple sclerosis is receiving baclofen. The nurse assessing the client monitors for which finding as an indication of a primary therapeutic response to the medication? a. decreased nausea b. decreased muscle spasms c. increased muscle tone and strength d. increased range of motion of all extremities

Answer: B Baclofen is a skeletal muscle relaxant and acts at the spinal cord level to decrease the frequency and amplitude of muscle spasms in clients with spinal cord injuries or diseases or with multiple sclerosis. Increased muscle tone and strength and increased range of motion of all extremities are not directly related to the effects of this medication. Decreased nausea is an incorrect option.

The home care nurse is visiting a client with a diagnosis of Parkinson's disease. The client is taking benztropine mesylate orally daily. The nurse provides information to the spouse regarding the side effects of this medication and should tell the spouse to report which side effect if it occurs? a. Shuffling gait b. Inability to urinate c. Decreased appetite d. Irregular bowel movements

Answer: B Benztropine mesylate is an anticholinergic, which causes urinary retention as a side effect. The nurse would instruct the client or spouse about the need to monitor for difficulty with urinating, a distended abdomen, infrequent voiding in small amounts, and overflow incontinence. The remaining options are unrelated to the use of this medication.

A client with multiple sclerosis is receiving diazepam, a centrally acting skeletal muscle relaxant. Which finding, if noted during the nursing assessment, would indicate that the client is experiencing a side/adverse effect of this medication? a. headache b. drowsiness c. urinary retention d. increased salivation

Answer: B Incoordination and drowsiness are common side/adverse effects of diazepam. The remaining options are unrelated to the use of this medication.

The nurse observes a patient with Parkinson's disease rocking side to side while sitting in the chair. Which action by the nurse is most appropriate? a. provide the patient with diversion activities b. document the activity in the patients health record c. take the patients blood pressure sitting and standing d. ask the patient if they are feeling either anxious or depressed.

Answer: B Patients with Parkinson's disease are taught to rock from side to side to stimulate balance mechanisms and decrease akinesia.

The clinic nurse is reviewing the record of a client scheduled to be seen in the clinic. The nurse notes that the client is taking selegiline hydrochloride. The nurse suspects that the client has which disorder? a. Diabetes mellitus b. Parkinson's disease c. Alzheimer's disease d. Coronary artery disease

Answer: B Selegiline hydrochloride is an antiparkinsonian medication. The medication increases dopaminergic action, assisting in the reduction of tremor, akinesia, and the rigidity of parkinsonism. This medication is not used to treat diabetes mellitus, Alzheimer's disease, or coronary artery disease.

The patient with a right-sided pleural effusion has stable vital signs and O2 at 6L/min via nasal cannula. A right-side chest tube is attached to straight drainage. Which actions would the nurse include in the plan of care? SATA a. placing the patient on NPO status b. administering analgesia as ordered c. maintaining high-Fowler's position d. encouraging deep breathing and coughing e. monitoring color and amount of chest tube drainage

Answer: B C D E Supportive care for a patient with a pleural effusion includes O2 therapy to treat hypoxemia, antipyretics (e.g., acetaminophen) for fever, and analgesics to relieve chest pain. Hydration, nutrition support, and positioning is part of the care plan. Work with the respiratory therapist to monitor the patient's condition and provide chest physiotherapy. Monitor chest tube drainage and output as appropriate for the care of the patient with a chest tube

While assessing the patient, the nurse observes constant bubbling in the water-seal chamber of the patients closed chest-drainage system. What should the nurse conclude? A) The system is functioning normally. B) The patient has a pneumothorax. C) The system has an air leak. D) The chest tube is obstructed.

Answer: C

You have just finished assisting the physician in removing Mr. H's chest tube. While securing the dressing, you notice that Mr. H is having difficulty breathing. Upon examination, you note that his trachea has shifted to the left side of his throat. You suspect that Mr. H has developed: A) early acute respiratory failure. B) aspiration of secretions. C) a pneumothorax. D) a normal finding after chest tube removal.

Answer: C

Baclofen is prescribed for the client with multiple sclerosis. The nurse determines that the medication is having the intended effect if which finding is noted in the client? a. increased muscle tone b. increased range of motion c. decreased muscle spasms d. decreased local pain and tenderness

Answer: C Baclofen is a skeletal muscle relaxant and acts at the spinal cord level to decrease the frequency and degree of muscle spasms in clients with multiple sclerosis, spinal cord injury, or other diseases. The other options are incorrect.

The nurse is preparing to ambulate a client with Parkinson's disease who has recently been started on levodopa/carbidopa. Before performing this activity with the client, the nurse should include which most important assessment in the client's plan of care? a. history of falls b. use of assistive devices c. postural (orthostatic) vital signs d. degree of exhibited intention tremor

Answer: C Clients with Parkinson's disease are at risk for postural (orthostatic) hypotension from the disease. This problem is exacerbated with the introduction of levodopa/carbidopa, which also can cause postural hypotension and increase the client's risk for falls. Although knowledge of the client's use of assistive devices and history of falls is helpful, neither of these options is the most important element of the assessment, based on the wording of this question. Clients with Parkinson's disease generally have resting tremor, not intention tremor.

A patient who has a right-sided chest tube following a thoracotomy has continuous bubbling in the suction-control chamber of the collection device. Which action by the nurse is most appropriate? a. Document the presence of a large air leak. b. Notify the surgeon of a possible pneumothorax. c. Take no further action with the collection device. d. Adjust the dial on the wall regulator to decrease suction.

Answer: C Continuous bubbling is expected in the suction-control chamber and indicates that the suction-control chamber is connected to suction. An air leak would be detected in the water-seal chamber. There is no evidence of pneumothorax. Increasing or decreasing the vacuum source will not adjust the suction pressure. The amount of suction applied is regulated by the amount of water in this chamber and not by the amount of suction applied to the system

The nurse is administering an intravenous dose of methocarbamol to a client with multiple sclerosis. For which adverse effect should the nurse monitor? a. tachycardia b. rapid pulse c. bradycardia d. hypertension

Answer: C Intravenous administration of methocarbamol can cause hypotension and bradycardia. The nurse needs to monitor for these adverse effects. Options 1, 2, and 4 are not effects with administration of this medication.

The nurse conducts a home visit for a patient with Parkinson disease taking levodopa/carbidopa daily. The patient has stopped eating meals with his family and has lost 3 pounds since the last visit a week ago. What action will the nurse take? a. plan 6 small high-protein meals a day b. provide information on a high-fat ketogenic diet. c. evaluate their ability with eating, swallowing, and use of assistive devices d. collaborate with the HCP about every other day levodopa/carbidopa dosing

Answer: C Patients with Parkinson's disorder have trouble with the coordinated act of eating, often require assistive devices to eat, and have difficulty swallowing. Diet is of major concern as malnutrition can occur without adequate calories and nutrient intake. The uncoordinated movements and tremors and slow eating can be embarrassing for the patient who may be isolating from the family. The nurse must determine if physical difficulty with the act of eating is causing the weight loss or if depression or another organic cause is to blame. Identifying the root of the problem would start with assessing the patient's abilities. Ketogenic diets are for headaches. Protein makes levodopa less effective, so protein should be limited to the evening meal. Reducing levodopa/carbidopa dosing will increase the tremors and bradykinesia.

The nurse is caring for a group of patients on a medical unit. After receiving report, which patient would the nurse see first? a. 42 yr w/ multiple sclerosis who was admitted with sepsis b. 72 yr w/ Parkinson's disease who has aspiration pneumonia c. 38 yr w/ myasthenia gravis who declined prescribed medications d. 45 yr w/ amyotrophic lateral sclerosis who refuses enteral feedings.

Answer: C Patients with myasthenia gravis who discontinue pyridostigmine (Mestinon) will develop myasthenic crisis. Myasthenia crisis results in severe muscle weakness and can lead to a respiratory arrest.

The nurse is caring for a group of patients on a medical unit. After receiving report, which patient would the nurse see first? a. 42 yr w/ multiple sclerosis who was admitted with sepsis b. 72 yr w/ Parkinson's disease who has aspiration pneumonia c. 38 yr w/ myasthenia gravis who declined prescribed medications. d. 45 yr w/ amyotrophic lateral sclerosis who refuses enteral feedings.

Answer: C Patients with myasthenia gravis who discontinue pyridostigmine (Mestinon) will develop myasthenic crisis. Myasthenia crisis results in severe muscle weakness and can lead to a respiratory arrest.

The student nurse diligently assesses her patient with a chest tube. She notices that the suction control chamber of the chest tube is not bubbling. What is the first thing this student should do? A) Document this normal finding B) Encourage the patient to cough and deep breathe C) Check the level of the suction on the wall D) Clamp the chest tube and call for help

Answer: C The level of suction is controlled by the amount of water in the suction control chamber. However, it would be prudent of the student nurse to check and see if the suction is even turned on. This portion of the chest tube should be gently bubbling, indicating the system is working. Coughing and deep breathing would not help turn the suction on. The student should never clamp the chest tube.

A nurse is caring for a client who is schedules to have his chest tube removed. Which of the following actions should the nurse take? a. cover the insertion site with a hydrocolloid dressing after removal. b. provide pain medication immediately after removal. c. instruct the client to perform the Valsalva maneuver during removal.' d. delegate removal of the chest tube to LPN.

Answer: C The nurse should instruct the client to perform the Valsalva maneuver during removal to maintain the appropriate amount of negative pressure in the chest in order to prevent air entry into the pleural space.

A nurse is planning care for a client following placement of a chest tube 1 hr ago. Which of the following actions should the nurse include in the plan of care? a. clamp the chest tube if there is continuous bubbling in the water seal chamber. b. keep chest tube drainage system at the lever of the right atrium. c. tape all connections between the chest tube and drainage system. d. empty the collection chamber and record the amount of drainage every 8 hr.

Answer: C The nurse should tape all connection's to ensure that the system is airtight and prevent the chest tubing from accidentally disconnecting.

The nurse is caring for a client with Parkinson's disease. Which finding about gait should the nurse expect to note in the client? a. walking on the toes b. unsteady and staggering c. shuffling and propulsive d. broad-based and waddling

Answer: C The parkinsonian gait is characterized by short, accelerating, shuffling steps. The client leans forward with the head, hips, and knees flexed and has difficulty starting and stopping. An ataxic gait is unsteady and staggering. A dystrophic gait is broad-based and waddling. Walking on the toes can occur from shortened Achilles tendons.

Which patient problem is the priority when caring for a patient with myasthenia gravis (MG)? a. acute confusion b. bowel incontinence c. activity intolerance d. disturbed sleep pattern

Answer: C The primary feature of MG is fluctuating weakness of skeletal muscle. Bowel incontinence and confusion are unlikely signs of MG. Although sleep disturbance is likely, activity intolerance is of primary concern.

When establishing a diagnosis of multiple sclerosis (MS), which diagnostic tests would the nurse expect? SATA a. EEG b. ECG c. CT scan d. Carotid duplex scan e. Evoked response testing f. Cerebrospinal fluid analysis

Answer: C E F No definitive diagnostic test exists for MS. Along with history and physical examination, CT scan, evoked response testing, cerebrospinal fluid analysis, and MRI are used to establish a diagnosis of MS. EEG, ECG, and carotid duplex scan are not used to diagnose MS.

A nurse is caring for a client who is postoperative following a thoracic lobectomy. The client has 2 chest tubes in place: 1 in the lower portion of the thorax and the other higher on the chest wall. When a family member asks why the client has 2 chest tubes, which of the following responses should the nurse make? a. two tubes were necessary due to excessive bleeding from the area of surgery. b. the tubes drain blood from 2 different lung areas. c. the lower tube will drain blood, and the higher tube will remove air. d. the second tube will take over if blood clots block the first tube.

Answer: C The tube that is lower on the thorax will drain blood, and the tube that is higher on the thorax will allow for removal of air.

New bubbling is observed in the water seal chamber after a patient with a pleural chest tube returns from a test. The nurse clamps the chest tube momentarily with a tubing clamp at the dressing site. When this is done, bubbling in the water seal stops. The next appropriate nursing action is to: A. Continue to monitor the water seal chamber for bubbling every hour for the next four hours B. Do nothing. This bubbling is normal in patients with pleural chest tubes C. Call the physician immediately and do not leave the patient's bedside because of the risk of respiratory failure D. Remove the chest tube dressing to see if one or more eyelets of the chest tube have been pulled out of the chest

Answer: D

A nurse is teaching about the adverse effects of baclofen with a client who has multiple sclerosis with spasms. Which of the following statements should the nurse identify as an indication that the client understands the teaching? a. Adverse effects include urinary frequency. b. I should increase my fiber intake to counteract the adverse effect of constipation. c. This medication can cause addiction. d. I should not stop taking this medication suddenly.

Answer: D Adverse effects associated with abrupt withdrawal of baclofen include visual hallucinations, paranoid ideations, and seizures.

The nurse sees the level of water in the water seal chamber rising very high. The nurse correlates which patient behavior with this rise? A) The patient is eating his lunch B) The patient is resting on his side C) The patient is squeezing the tubing D) The patient is coughing viciously

Answer: D Coughing, sneezing or other forces can cause an increase in negative pressure which will in turn cause an increase in the water in the water seal chamber. Eating or resting should not affect the negative pressure in the tube. Squeezing, kinking, or somehow cutting off the flow into the chest tube would increase positive pressure, not negative.

A nurse is providing teaching to a client who has multiple sclerosis and a new prescription for baclofen PO. Which of the following pieces of information should the nurse include? a. You should take this on an empty stomach to increase absorption. b. You can stop taking it once your back spasms disappear. c. You can expect to experience urinary frequency when you first start taking it. d. You should change positions slowly while on this medication.

Answer: D Dizziness and hypotension are adverse effects of this medication. Baclofen should be taken with milk or food to minimize GI stress. Stopping medication abruptly can cause withdrawal reaction like hallucinations and increased spasticity. Urinary frequency is an adverse effect of medication and should be reported to provider.

Carbidopa-levodopa is prescribed for a client with Parkinson's disease. The nurse monitors the client for side and adverse effects of the medication. Which finding indicates that the client is experiencing an adverse effect? a. Pruritus b. Tachycardia c. Hypertension d. Impaired voluntary movements

Answer: D Dyskinesia and impaired voluntary movements may occur with high carbidopa-levidopa dosages. Nausea, anorexia, dizziness, orthostatic hypotension, bradycardia, and akinesia are frequent side effects of the medication.

Carbidopa-levodopa is prescribed for a client with Parkinson's disease. The nurse monitors the client for side and adverse effects of the medication. Which finding indicates that the client is experiencing an adverse effect? a. pruritus b. tachycardia c. hypertension d. impaired voluntary movements

Answer: D Dyskinesia and impaired voluntary movements may occur with high carbidopa-levodopa dosages. Nausea, anorexia, dizziness, orthostatic hypotension, bradycardia, and akinesia are frequent side effects of the medication

The patient has a chest tube on the ride side. What intervention would the nurse include in the plan of care to prevent complications? a. Maintain total bed rest. b. Place chest tube system above the chest. c. Strip the chest tube tubing every 2-4 hours. d. Encourage use of incentive spirometer every hour while awake.

Answer: D Encourage use of incentive spirometry every hour while awake to prevent atelectasis or pneumonia. Have the patient breathe deeply periodically to promote lung expansion. Encourage ROM exercises to the shoulder on the affected side Keep all tubing loosely coiled below chest level. Position tubing so that drainage flows freely. Never raise the drainage system to the level of the patient's chest because fluid can drain back into the lungs. Never compress, milk, or strip chest tube tubing

The nurse is caring for a client with an exacerbation of multiple sclerosis. Which medication(s) will the nurse expect to be prescribed to hasten recovery from the exacerbation? a. Carbamazepine and phenytoin by mouth b. Lioresal by mouth and diazepam intravenously c. Phenytoin intravenously, then tapered to oral route d. Methylprednisolone and cyclophosphamide intravenously

Answer: D Intravenous methylprednisolone or adrenocorticotropic hormone in combination with cyclophosphamide may be prescribed to accelerate recovery from an exacerbation of multiple sclerosis. Carbamazepine may be prescribed for trigeminal neuralgia, and phenytoin may be prescribed to control seizures. Lioresal and diazepam are used to treat muscle spasticity.

A client comes to the emergency department in severe respiratory distress following left-sided blunt chest trauma. The nurse notes absent breath sounds on the client's left side and a tracheal shift to the right. For which of the following procedures should the nurse prepare the client? a. tracheostomy placement b. thoracentesis c. CT scan of chest d. chest tube insertion

Answer: D Manifestations are that of pneumothorax due to blunt chest trauma. The nurse should prepare for the provider to insert a chest tube and connect it to a water-seal drainage system. A thoracentesis is indicated for a client who has increase of pleural fluid due to cancer, pleurisy, or TB.

The nurse is assigned to care for a client with multiple sclerosis who is receiving an intravenous dose of methocarbamol. The nurse monitors the client knowing that which is an expected side effect? a. insomnia b. excitability c. hypertension d. dark green-colored urine

Answer: D Methocarbamol is a skeletal muscle relaxant. It may cause the urine to turn a brown, black, or dark green color, and the client needs to be told that this is a harmless effect. This medication can cause hypotension. Drowsiness and dizziness can also occur. Therefore, the remaining options are incorrect.

A patient with a diagnosis of Parkinson's disease (PD) is admitted to a long-term care facility. Which action would the health care team take to promote adequate nutrition? a. Administer multivitamins every morning and with each meal. b. Provide a diet that is low in complex carbohydrates and high in protein. c. Give the patient with a pureed diet that is high in potassium and low in sodium. d. Provide small, frequent meals throughout the day that are easy to chew and swallow.

Answer: D Nutrition support is a priority in the care of persons with PD. Patients may benefit from smaller, more frequent meals that are easy to chew and swallow. Multivitamins are not necessary at each meal. Vitamin and protein intake must be monitored to prevent interactions with medications. Introducing a minced or pureed diet is likely premature, and a low carbohydrate diet is not indicated.

A client with Parkinson's disease has begun therapy with levodopa/carbidopa. The nurse determines that the client understands the action of the medication if he or she verbalizes that results may not be apparent for how long? a. 1 week b. 24 hours c. 2-3 days d. 2-3 weeks

Answer: D Signs and symptoms of Parkinson's disease usually begin to resolve within 2 to 3 weeks of starting therapy, although in some clients marked improvement may not be seen for up to 6 months. The client needs to understand this concept to aid in compliance with medication therapy.

The nurse is instructing a client with Parkinson's disease about preventing falls. Which client statement reflects a need for further teaching? a. "I can sit down to put on my pants and shoes." b. "My son removed all loose rugs from my bedroom." c. "I try to exercise every day and rest when I'm tired." d. "I don't need to use my walker to get to the bathroom."

Answer: D The client with Parkinson's disease should be instructed regarding safety measures in the home. The client should use his or her walker as support to get to the bathroom because of bradykinesia. The client should sit down to put on pants and shoes to prevent falling. The client should exercise every day in the morning when energy levels are highest. The client should have all loose rugs in the home removed to prevent falling.

A nurse is caring for a client who has a chest tube. The nurse notes that the chest tube has become disconnected from the chest drainage system. Which of the following actions should the nurse take? a. place drainage system at the head of the client's bed b. increase suction to the chest drainage system c. place the client on low-flow oxygen via nasal cannula d. immerse the end of the chest tube in a bottle of sterile water

Answer: D To prevent a pneumothorax from developing, a temporary water seal can be established by immersing the end of the chest tube in an open bottle of sterile water. This allows air to escape and not enter the pleural space.

A nurse is providing postoperative care for a client who has 2 chest tubes in place following a lobectomy. The client asks the nurse the reason for having 2 chest tubes. The nurse should inform the client that the lower chest tube is placed for which of the following reasons? a. removing air from the pleural space b. creating access for irrigating the chest cavity c. evacuating secretions from the bronchioles and alveoli d. draining blood and fluid from the pleural space

Answer: D The nurse should inform the client that blood and fluids tend to accumulate in the bases and posterior areas of the pleural cavity following a lobectomy. For this reason, the lower chest tube primarily drains blood and fluid from the pleural space.

Which of the following is a potential complication of chest tube insertion? a. Hypotension b. Hypertension c. Bradycardia d. Tachycardia

Answer: a. Hypotension Explanation: Hypotension is a potential complication of chest tube insertion. The procedure can cause a sudden drop in blood pressure due to the release of pressure in the pleural space.

Which medication is used to help loosen and thin secretions in patients with a chest tube? a. Albuterol b. Dornase alfa c. Guaifenesin d. Acetylcysteine

Answer: d. Acetylcysteine Explanation: Acetylcysteine is a mucolytic agent that helps loosen and thin secretions in patients with a chest tube. It works by breaking down the chemical bonds between the mucus molecules, making them less viscous and easier to expectorate. Albuterol is a bronchodilator that can help open up the airways, but it is not used specifically for chest tube care. Dornase alfa is a medication used for cystic fibrosis and does not have a role in chest tube care. Guaifenesin is an expectorant that may help thin secretions, but it is not as effective as acetylcysteine.

What is the medication of choice for pain management during chest tube insertion? a. Morphine b. Fentanyl c. Ketamine d. Lidocaine

Answer: d. Lidocaine Explanation: Lidocaine is the medication of choice for pain management during chest tube insertion. It is a local anesthetic that can be administered topically or subcutaneously. It can provide rapid and effective pain relief for the patient during the procedure. Morphine and fentanyl are potent opioid analgesics that may be used for pain management, but they are not typically used during chest tube insertion. Ketamine is a dissociative anesthetic that may be used for pain management, but it is not commonly used for chest tube insertion.

A nurse is caring for several clients in an extended care facility. Which of the following clients is the highest priority to observe during meals? A. A client who has decreased vision B. A client who has Parkinson's disease C. A client who has poor dentition D. A client who has anorexia

B. CORRECT: A client who has Parkinson's disease is at risk for aspiration. Due to this safety risk, this client is the highest priority to observe during meals.

A nurse is assessing a client who has a chest tube and drainage system in place. Which of the following are expected findings? (Select all that apply.) A. Continuous bubbling in the water seal chamber B. Gentle constant bubbling in the suction control chamber C. Rise and fall in the level of water in the water seal chamber with inspiration and expiration D. Exposed sutures without dressing E. Drainage system upright at chest level

B. CORRECT: Gentle bubbling in the suction control chamber is an expected finding as air is being removed. C. CORRECT: A rise and fall of the fluid level in the water seal chamber upon inspiration and expiration indicates that the drainage system is functioning properly.

A nurse is caring for a client who has multiple sclerosis. Which of the following findings should the nurse expect? A. Fluctuations in blood pressure B. Loss of cognitive function C. Ineffective cough D. Drooping eye lids

B. CORRECT: Loss of cognitive function is a manifestation associated with MS. A. Fluctuations in blood pressure is a manifestation associated with amyotrophic lateral sclerosis. C. Ineffective cough is a manifestation associated with amyotrophic lateral sclerosis. D. Drooping eyelids is a manifestation associated with myasthenia gravis.

A nurse is developing a plan of care for the nutritional needs of a client who has stage IV Parkinson's disease. Which of the following actions should the nurse include? (Select all that apply.) A. Provide three large balanced meals daily. B. Record diet and fluid intake daily. C. Document weight every other week. D. Offer cold fluids such as milkshakes. E. Offer nutritional supplements between meals.

B. CORRECT: Record the client's diet and fluid intake daily to assess for dietary needs and to maintain adequate nutrition and hydration. D. CORRECT: Provide cold fluids such as milkshakes. Thick and cold fluids are tolerated easier by the client. E. CORRECT: Offer nutritional supplements between meals to maintain the client's weight.

A nurse is assessing a client for manifestations of Parkinson's disease. Which of the following are expected findings? (Select all that apply.) A. Decreased vision B. Pill rolling tremor of the fingers C. Shuffling gait D. Drooling E. Bilateral ankle edema F. Lack of facial expression

B. CORRECT: The client who has PD can manifest pill rolling tremors of the fingers due to overstimulation of the basal ganglia by acetylcholine, making controlled movement difficult. C. CORRECT: The client who has PD can manifest shuffling gait because of overstimulation of the basal ganglia by acetylcholine, making controlled movement difficult. D. CORRECT: The client who has PD can manifest drooling because of overstimulation of the basal ganglia by acetylcholine, making the controlled movement of swallowing secretions difficult. F. CORRECT: The client who has PD can manifest a lack of facial expressions due to overstimulation of the basal ganglia by acetylcholine, making controlled movement difficult.

A nurse is teaching a client who has angina pectoris and is learning how to treat acute anginal attacks. The clients asks, "What is my next step if I take one tablet, wait 5 minutes, but still have anginal pain?" Which of the following responses should the nurse make? A. "Take two more sublingual tablets at the same time." B. "Call the emergency response team." C. "Take a sustained release nitroglycerin capsule." D. "Wait another 5 minutes then take a second sublingual tablet."

B. CORRECT: The next step is to call 911 and then take a second sublingual tablet. If the first tablet does not work, the client might be having a myocardial infarction. The client can take a third tablet if the second one has not relieved the pain after waiting an additional 5 minutes.

A nurse is caring for a client who has a chest tube and drainage system in place. The nurse observes that the chest tube was accidentally removed. Which of the following actions should the nurse take first? A. Obtain a chest x-ray. B. Apply sterile gauze to the insertion site. C. Place tape around the insertion site. D. Assess respiratory status.

B. CORRECT: Using the airway, breathing, and circulation (ABC) priority setting framework, application of a sterile gauze to the site should be the first action for the nurse to take. This allows air to escape and reduces the risk for development of a tension pneumothorax

A nurse is presenting a community education program on recommended lifestyle changes to prevent angina and myocardial infarction. Which of the following changes should the nurse recommend be made first? A. Diet modification B. Relaxation exercises C. Smoking cessation D. Taking omega 3 capsules

C. CORRECT: According to the airway, breathing, and circulation (ABC) priority setting framework, the first step is to recommend the clients to stop smoking. Nicotine causes vasoconstriction, elevates blood pressure, and narrows coronary arteries.

A nurse on a cardiac unit is reviewing the laboratory findings of a client who has a diagnosis of myocardial infarction (MI) and reports that his dyspnea began 2 weeks ago. Which of the following cardiac enzymes would confirm the MI occurred 14 days ago? A. CK MB B. Troponin I C. Troponin T D. Myoglobin

C. CORRECT: The Troponin T level will still be evident 10 to 14 days following an MI

A nurse is teaching a client who has a new prescription for levodopa/carbidopa for Parkinson's disease. Which of the following instructions should the nurse include? A. Increase intake of protein rich foods. B. Expect muscle twitching to occur. C. Take this medication with food. D. Anticipate relief of manifestations in 24 hr.

C. CORRECT: The client should take this medication with food to reduce GI effects.

A nurse is caring for a client who displays manifestations of stage III Parkinson's disease. Which of the following actions should the nurse include? A. Recommend a community support group. B. Integrate a daily exercise routine. C. Provide a walker for ambulation. D. Perform ADLs for the client.

C. CORRECT: The client should use a walker for ambulation in stage III of Parkinson's disease because movement slows down significantly and gait disturbances occur.

A nurse is teaching a client who has multiple sclerosis and a new prescription for baclofen. Which of the following statements should the nurse include in the teaching? A. "This medication will help you with your tremors." B. "This medication will help you with your bladder function." C. "This medication can cause your skin to bruise easily." D. "This medication can cause you to experience dizziness."

D. CORRECT: Baclofen is an antispasmodic medication that is given to clients who have MS to treat muscle spasms. An adverse effect of this medication is drowsiness, as well as dizziness. Instruct the client to monitor for these findings, as they can lead to impaired safety. The client should be instructed not to discontinue baclofen abruptly

A nurse is caring for a client who has Parkinson's disease and is starting to display bradykinesia. Which of the following is an appropriate action by the nurse? A. Teach the client to walk more quickly when ambulating. B. Complete passive range of motion exercises daily. C. Place the client on a low protein, low calorie diet. D. Give the client extra time to perform activities.

D. CORRECT: Bradykinesia is abnormally slowed movement and is seen in clients who have PD. The client should be given extra time to perform activities and should be encouraged to remain active.

A nurse is assisting a provider with the removal of a chest tube. Which of the following actions should the nurse take? A. Instruct the client to lie prone with arms by the sides. B. Complete a surgical checklist on the client. C. Remind the client that there is minimal discomfort during the removal process. D. Place an occlusive dressing over the site once the tube is removed.

D. CORRECT: The nurse should place an occlusive dressing over the site once the tube is removed and observe the site for drainage


Related study sets

Chapter 44 Pilliterri Family Final HEME

View Set

12. Entorno turístico lección 11: "Un viaje por carretera" (pg. 86 a 89)

View Set

'What is the Horror Genre?' Study Guide

View Set

Module 1, Unit 4 - Incident Response

View Set

BA 325 Final Exam Homework questions

View Set

NS ch 38 the solar system review questions

View Set

Canvas Quiz: Gastrointestinal (GI) Bleed

View Set